Difference between revisions of "1985 AJHSME Problems/Problem 21"

(Solution)
(Solution)
Line 7: Line 7:
 
==Solution==
 
==Solution==
  
Assume his salary is 100 dollars then the next year he would have 110 dollars then the next year he would have 121 dollars then the next year he would have 133.1 dollars so therefore it is A
+
Assume his salary is 100 dollars then the next year he would have 110 dollars then the next year he would have 121 dollars then the next year he would have 133.1 dollars then, the last year, he would have 164.41; therefore the answer is <math>\boxed{\text{E}}</math>.
 
 
so the percent is smaller than <math>40\% </math> and the only choice left is <math>\boxed{\text{A}}</math>.
 
It is 4 raises, not 3.
 
Thus, the correct answer is a 46.41 percent increase, or <math>\boxed{\text{E}}</math>.
 
  
 
==See Also==
 
==See Also==

Revision as of 18:58, 22 December 2015

Problem

Mr. Green receives a $10\%$ raise every year. His salary after four such raises has gone up by what percent?

$\text{(A)}\ \text{less than }40\% \qquad \text{(B)}\ 40\% \qquad \text{(C)}\ 44\% \qquad \text{(D)}\ 45\% \qquad \text{(E)}\ \text{more than }45\%$

Solution

Assume his salary is 100 dollars then the next year he would have 110 dollars then the next year he would have 121 dollars then the next year he would have 133.1 dollars then, the last year, he would have 164.41; therefore the answer is $\boxed{\text{E}}$.

See Also

1985 AJHSME (ProblemsAnswer KeyResources)
Preceded by
Problem 20
Followed by
Problem 22
1 2 3 4 5 6 7 8 9 10 11 12 13 14 15 16 17 18 19 20 21 22 23 24 25
All AJHSME/AMC 8 Problems and Solutions


The problems on this page are copyrighted by the Mathematical Association of America's American Mathematics Competitions. AMC logo.png